0 Daumen
111 Aufrufe

Aufgabe:

Es geht um Aufgabe 2b)(i)

leider beim letzten Lösungsschritt Unklarheit, d.h. was soll eigentlich ln(|e^iphi|) bedeuten, bzw wie kann man dies in dem Fall umschreiben. Teillösung ist hochgeladen und würde mich freuen falls jemand helfen könnte.Screenshot (457).png

Text erkannt:

1. Aufgabe: Komplexer Logarithmus

Die Umkehrfunktion von
\( \begin{aligned} \{z \in \mathbb{C} \mid-\pi<\operatorname{Im}(z)<\pi\} & \longrightarrow \mathbb{C} \backslash\{x \in \mathbb{R} \mid x \leq 0\} \\ z & \longmapsto e^{z} \end{aligned} \)
heißt Hauptzweig des komplexen Logarithmus und wird im Folgenden mit log beze
\( \log : \mathbb{C} \backslash\{x \in \mathbb{R} \mid x \leq 0\} \longrightarrow\{z \in \mathbb{C} \mid-\pi<\operatorname{Im}(z)<\pi\} . \)
a) Zeigen Sie, dass \( \log \) eine konforme Abbildung mit \( \log ^{\prime}(z)=\frac{1}{z} \).
b) Zeigen Sie (rechnerisch oder geometrisch), dass gilt:
(i) \( \log z=\ln |z|+i \arg z \) für \( z \in \mathbb{C} \backslash\{x \in \mathbb{R} \mid x \leq 0\} \).
(ii) \( \arg z=\frac{1}{2 i} \log \frac{z}{\bar{z}} \) für \( z \in \mathbb{C} \backslash\{x \in \mathbb{R} \mid x \leq 0\} \).
(iii) \( \arctan x=\arg (1+i x)=\frac{1}{2 i} \log \frac{1+i x}{1-i x} \) für \( x \in \mathbb{R} \).
Hierbei bezeichnet \( \arg z \) das Argument von \( z \) mit \( -\pi<\arg z<\pi \).


Problem/Ansatz:

Untitled - 2024-06-25T105126.852.jpg

Text erkannt:

(ii)
\( \arg z=\frac{1}{2 i} \log \frac{z}{\bar{z}} \operatorname{lin} z \in(\backslash\{x \in \mathbb{R} \mid x \leq 0\} \)
\( \begin{aligned} & \ln \left(1-e^{i \varphi}\right)-\ln \left(1+|| e^{i \varphi} \mid\right) \cdot \frac{1}{2 i} \\ = & \ln (r)+i \varphi-\left(\ln r+\ln \left(\left|e^{i \varphi}\right|\right)\right. \\ = & \end{aligned} \)

LG

Avatar von

Hallo
da |eiφ|=1  ist ln(1)=0
Gruß lul

Ich verstehe das Aufgeschriebene nicht. Ich verstehe auch die Aufgabe nicht: Für \(z=i\) ist \(z/\bar{z}=-1\), also wäre die rechte Seite in der Formel nicht definiert??

1 Antwort

0 Daumen

Wer auch immer die Aufgabe gestellt hat, hat offenbar vergessen, dass im Komplexen \(\log (zw) \neq log z + \log w\) passieren kann.

Bsp.: \(z= e^{\frac 34\pi i},\, w= e^{\frac 12 \pi i}\) (gern selber nachrechnen oder schnell sich geometrisch klarmachen)

Die Regel (ii) sollte daher eher so lauten:

\(\arg z = \frac 1{2i}\left(\log z +\log \frac 1{\bar z}\right) = \frac 1{2i}\left(\log z -\log \bar z\right)\)

Das kannst du jetzt schnell mit (i) zeigen.


Dein Aufschrieb stellt momentan keine Teillösung dar. Versuch es nochmal in Ruhe und mit Überlegung. Nutze dabei

\(z = |z|e^{i\cdot \arg z}\) und \( \bar z = |\bar z|e^{i\cdot \arg \bar z} = |z|e^{-i\cdot \arg z}\)

Avatar von 10 k

Ein anderes Problem?

Stell deine Frage

Willkommen bei der Mathelounge! Stell deine Frage einfach und kostenlos

x
Made by a lovely community